[seqfan] A MathOverflow question related to A000081

Vladimir Reshetnikov v.reshetnikov at gmail.com
Tue Feb 26 19:52:30 CET 2019


I asked a question at MathOverflow related to https://oeis.org/A000081:
https://mathoverflow.net/questions/324203/integrals-of-power-towers

Please share your thoughts there or in this mailing list.

--
Best regards
Vladimir Reshetnikov



More information about the SeqFan mailing list